Real analysis: prove the limit exists

In summary, the proofs for the three given limits show that they all exist at their respective values. For the first limit, the chosen delta is dependent on x, which is not ideal, but the proof itself is sound. For the second limit, the proof shows that the limit does not equal 2, by showing that it exists elsewhere. For the third limit, a transformation is used to simplify the limit and a delta is chosen based on the given epsilon, proving that the limit exists at 1. Additionally, a shortcut can be used to prove the existence of a limit for any function in the real numbers, as well as for the sum and product of two functions with existing limits at a given point.
  • #1
docnet
Gold Member
696
348
Homework Statement
Prove the limit exists.
Relevant Equations
##\text{lim}_{x\rightarrow a}\quad f=L## means ##\forall \epsilon>0, \exists \quad \delta ## such that ##|x-a|<\delta \Rightarrow |f(x)-L|<\epsilon##.
Prove that each of the limits exists or does not exist.

1. ##\text{lim}_{x\rightarrow 2}(x^2-1)=3##

##\text{lim}_{x\rightarrow 2}(x^2-1)=3## if ##\forall \epsilon>0, \exists \delta ## such that ##|x-2|<\delta \Rightarrow |f(x)-3|<\epsilon##.
\begin{align}&|x^2-1|=|x+1||x-1|\leq \epsilon\\
\Rightarrow &|x-1|\leq \frac{\epsilon}{|x+1|}\\
&\delta=\frac{\epsilon}{|x+1|}\end{align}

2. ##\text{lim}_{x\rightarrow 1}(2x-1)\neq 2##
##\text{lim}_{x\rightarrow 1}(2x-1)\neq 2## if the limit exists elsewhere than ##2##. This means for all ##\epsilon>0##, there exists a ##\delta## such that ##|x-1|<\delta## implies ##\text{lim}_{x\rightarrow 1}(2x-1)= L## for some ##L\neq 2##.
\begin{align}&|2x-1-1|=2|x-1|\leq \epsilon\Rightarrow |x-1|\leq \frac{\epsilon}{2}\\
&\delta=\frac{\epsilon}{|2|}\end{align}

3. ##\text{lim}_{x\rightarrow \infty} (1+\frac{1}{n+1})=1##
We use ##x\rightarrow \frac{1}{x}## to transform the limit.
\begin{align}&\text{lim}_{x\rightarrow \infty} (1+\frac{1}{n+1})\rightarrow \text{lim}_{x\rightarrow 0} (1+\frac{n}{n+1})=1\\
&|1+\frac{n}{n+1}-1| \leq |n| <\epsilon \Rightarrow |n|\leq \epsilon\\
&\delta=\epsilon\\
&\therefore \forall \epsilon>0, |n|<\epsilon \Rightarrow |1+\frac{n}{n+1}-1|<\epsilon\\
&\therefore \forall \epsilon>0, |n-\infty|<\epsilon \Rightarrow |1+\frac{1}{n+1}-1|<\epsilon\\
\end{align}
 
Physics news on Phys.org
  • #2
For number 1, your delta depends on x, which is generally not good practice. For number 2 I'm not even sure what you're trying to do. Are you trying to prove a limit exists? For number 3 there's a lot of confusion between x and n, which makes it hard to tell what's going on.I think it would help to take a step back and try to write the proof only in the "right" direction. For #1, start off by telling us what ##\delta## you want to pick for a given ##\epsilon##, instead of writing down the steps you used to derive it (those steps can be useful for figuring out what you want the proof to look like, but they are not the actual proof typically).
 
  • Like
Likes PeroK and docnet
  • #3
Office_Shredder said:
For number 1, your delta depends on x, which is generally not good practice. For number 2 I'm not even sure what you're trying to do. Are you trying to prove a limit exists? For number 3 there's a lot of confusion between x and n, which makes it hard to tell what's going on.I think it would help to take a step back and try to write the proof only in the "right" direction. For #1, start off by telling us what ##\delta## you want to pick for a given ##\epsilon##, instead of writing down the steps you used to derive it (those steps can be useful for figuring out what you want the proof to look like, but they are not the actual proof typically).
Okay, I understand.
 
  • #4
1)

##\text{lim}_{x\rightarrow 2}(x^2-1)=3## if ##\forall \epsilon>0, \exists \delta ## such that ##|x-2|<\delta \Rightarrow |f(x)-3|<\epsilon##.

Let ##\delta<1##.

Then, ##|x-2|<\delta<1\Rightarrow |x-2|<1 \Rightarrow -1<x-2<1##.

Adding 4 to the last inequality gives
##3<x+2<5\Rightarrow |x+2|<5##.

Then,
##|x^2-4|=|x+2||x-2|<5|x-2|<\epsilon\Rightarrow |x-2|<\epsilon/5##.

Select ##\delta=\text{min}(1,\epsilon/5)##.

Check: ##\forall \epsilon>0,\delta=\text{min}(1,\epsilon/5)##.
Then ##|x^2-4|=|x+2||x-2|<5|x-2|<5\delta\leq 5\cdot \epsilon/5=\epsilon##.
 
Last edited:
  • #5
docnet said:
1)

##\text{lim}_{x\rightarrow 2}(x^2-1)=3## if ##\forall \epsilon>0, \exists \delta ## such that ##|x-2|<\delta \Rightarrow |f(x)-3|<\epsilon##.

Let ##\delta<1##.

Then, ##|x-2|<\delta<1\Rightarrow |x-2|<1 \Rightarrow -1<x-2<1##.

Adding 4 to the last inequality gives
##3<x+2<5\Rightarrow |x+2|<5##.

Then,
##|x^2-4|=|x+2||x-2|<5|x-2|<\epsilon\Rightarrow |x-2|<\epsilon/5##.

Select ##\delta=\text{min}(1,\epsilon/5)##.

Check: ##\forall \epsilon>0,\delta=\text{min}(1,\epsilon/5)##.
Then ##|x^2-4|=|x+2||x-2|<5|x-2|<5\delta\leq 5\cdot \epsilon/5=\epsilon##.
This is not bad. Although, you seem to have switched from ##x^2 -1## to ##x^2 - 4## at some point!

The proof proper should start with "Let ##\epsilon > 0##". In any case, you introduce ##\epsilon## without saying what it is. At that point the proof gets a little confused between the proof itself and some supporting notes/calculations.
 
  • Like
Likes docnet
  • #6
PeroK said:
This is not bad. Although, you seem to have switched from ##x^2 -1## to ##x^2 - 4## at some point!
***sweats profusely***o:)

Question: Does one always include the supporting notes (like the derivation of ##\delta##) as part of the proof?3)

Supporting notes:
$$\displaystyle{\lim_{n \to \infty}}\Big(1+\frac{1}{n+1}\Big)=1\iff \displaystyle{\lim_{x \to 0}}\Big(1+\frac{x}{x+1}\Big)=1$$
Let ##\delta<\frac{1}{2}##.

Then,
$$|x|<\delta<\frac{1}{2}\Longrightarrow |x|<\frac{1}{2}$$
$$\Longrightarrow |x+1|<\frac{3}{2}$$
$$\Longrightarrow \frac{x}{ |x+1|}<\frac{3|x|}{2}$$
Select ##\delta = \text{min}(\frac{1}{2}, \frac{3\epsilon}{2})##.

The proof:

Let ##\epsilon>0## be given. ##\displaystyle{\lim_{x \to 0}}\Big(1+\frac{x}{x+1}\Big)=1## means there exists a ##\delta## for every ##\epsilon>0## such that
$$|x|<\delta\Longrightarrow |\frac{x}{x+1}|<\epsilon$$
Choose ##\delta = \text{min}(\frac{1}{2}, \frac{3\epsilon}{2})##.
$$|\frac{x}{x+1}<\frac{2|x|}{3}$$
Since if ##|x|< \frac{1}{2}, |x+1|<\frac{3}{2}## and
$$\frac{2|x|}{3}<\frac{2\delta}{3}<\frac{2}{3}\cdot\frac{3\epsilon}{2}=\epsilon$$
 
  • #7
Theres also somewhat of a short cut for this types of problems involving f(x)=x. Prove that the limit of f(x) exist at any point in R. Then prove that if two functions have a limit at a point c, then their sum and product also have a limit at that point. .
 
  • Like
Likes docnet
  • #8
Docnet, I think the best way to start #1 is.

Let ##\epsilon > 0##, and let ##\delta =\min(0.9,\epsilon/6)##

(I picked 0.9 because your first step assumed ##\delta## was strictly less than 1! Similar for ##\epsilon/6## instead of 5)Then, because ##\delta<1##, blah blah blah (eq. 1)

Since ##\delta < \epsilon/5##, blah blah blah (eq 2).

Combining (1) and (2) yields [stuff]

3 is ok, but I wouldn't call it "supporting notes". Feel free to call it lemma #1, and then below say by lemma #1, we get the following. (Also note again your ##\delta## is slightly too large because of strict vs weak inequalities. If only Leibniz had defined continuity using weak inequalities the world would be a better place)
 
Last edited by a moderator:
  • Like
  • Informative
Likes PeroK and docnet
  • #9
@docnet I would be comfortable with the idea of intepreting the definition of a limit in terms of what needs to be shown in a particular case. For example:

(*) Let ##\epsilon > 0##; choose ##\delta ...##, then ##0 < |x - a| < \delta \Rightarrow |f(x) - L| < \epsilon##.

Once you have that you can conclude that ##\lim_{x \rightarrow a} f(x) = L##. That is enough.

You don't have to reconstruct a pseudo-definition of the limit with ##\forall \epsilon > 0 ...##.

What's going on here might be an interesting question for the foundations of mathematics, but as far as real analysis is concerned it is enough to show (*).
 
  • Like
Likes docnet and SammyS
  • #10
docnet said:
Prove that each of the limits exists or does not exist.

1. ##\displaystyle \lim_{x\rightarrow 2}(x^2-1)=3##

2. ##\displaystyle \lim_{x\rightarrow 1}(2x-1)\neq 2##

3. ##\displaystyle \lim_{n\rightarrow \infty} (1+\frac{1}{n+1})=1##
I see that so far, this thread mostly addresses Problem 1.

I suggest that you post Problems 2 and 3 each in a thread of its own.
 
Last edited:
  • #11
PeroK said:
@docnet I would be comfortable with the idea of intepreting the definition of a limit in terms of what needs to be shown in a particular case. For example:

(*) Let ##\epsilon > 0##; choose ##\delta ...##, then ##0 < |x - a| < \delta \Rightarrow |f(x) - L| < \epsilon##.

Once you have that you can conclude that ##\lim_{x \rightarrow a} f(x) = L##. That is enough.

You don't have to reconstruct a pseudo-definition of the limit with ##\forall \epsilon > 0 ...##.

What's going on here might be an interesting question for the foundations of mathematics, but as far as real analysis is concerned it is enough to show (*).
Thank you, I understand. Hopefully, I did better this time.

2)
Prove that ##\displaystyle{\lim_{x \to 1}}(2x-1)\neq 2##.

Method 1:
Given ##\epsilon>0##, choose ##\delta=\epsilon/2##.
Then,
$$|x-1|<\delta\Longrightarrow |x-1|<\epsilon/2\Longrightarrow 2|x-1|<\epsilon$$
$$2|x-1|<\epsilon\Longrightarrow |2x-1-1|<\epsilon$$
So $$1=\displaystyle{\lim_{x \to 1}}(2x-1)\neq 2$$.

Method 2:
$$\neg \Big(\forall\epsilon>0, \exists \delta>0\quad \text{s.t.}\quad|x-1|<\delta\Longrightarrow |2x-1-2|<\epsilon \Big)$$
$$\equiv\exists \epsilon>0\quad\text{s.t.}\quad \forall \delta>0, \exists x\quad\text{s.t.}\quad (|x-1|<\delta)\land(|2x-1-2|\geq \epsilon)$$

Given ##\delta>0##, choose ##\epsilon=1## and ##x=1##.

$$|2x-1-2|\geq \epsilon\Longrightarrow |2\cdot 1-3|=|-1|=1$$
and
$$|x-1|<\delta \Longrightarrow |1-1|=0<\delta$$
By proving the negated statement, ##\displaystyle{\lim_{x \to 1}}(2x-1)\neq 2##
 
Last edited:
  • #12
MidgetDwarf said:
Theres also somewhat of a short cut for this types of problems involving f(x)=x. Prove that the limit of f(x) exist at any point in R. Then prove that if two functions have a limit at a point c, then their sum and product also have a limit at that point. .
This is truly useful information! Although, the professor told us we have to use the epsilon-delta definition of limit for these problems.
SammyS said:
I see that so far, this thread mostly addresses Problem 1.

I suggest that you post Problems 2 and 3 each in a thread of its own.
After post #11, I hope that all three problems are almost all solved.
Office_Shredder said:
3 is ok, but I wouldn't call it "supporting notes". Feel free to call it lemma #1, and then below say by lemma #1, we get the following. (Also note again your ##\delta## is slightly too large because of strict vs weak inequalities. If only Leibniz had defined continuity using weak inequalities the world would be a better place)
Thank you, I understand. Why did Leibniz not define continuity using weak inequalities? (really curious).
 
  • #13
docnet said:
Thank you, I understand. Hopefully, I did better this time.

2)
Prove that ##\displaystyle{\lim_{x \to 1}}(2x-1)\neq 2##.

Method 1:
Given ##\epsilon>0##, choose ##\delta=\epsilon/2##.
Then,
$$|x-1|<\delta\Longrightarrow |x-1|<\epsilon/2\Longrightarrow 2|x-1|<\epsilon$$
$$2|x-1|<\epsilon\Longrightarrow |2x-1-1|<\epsilon$$
So $$1=\displaystyle{\lim_{x \to 1}}(2x-1)\neq 2$$.

Method 2:
$$\neg \Big(\forall\epsilon>0, \exists \delta>0\quad \text{s.t.}\quad|x-1|<\delta\Longrightarrow |2x-1-2|<\epsilon \Big)$$
$$\equiv\exists \epsilon>0\quad\text{s.t.}\quad \forall \delta>0, \exists x\quad\text{s.t.}\quad (|x-1|<\delta)\land(|2x-1-2|\geq \epsilon)$$

Given ##\delta>0##, choose ##\epsilon=1## and ##x=1##.

$$|2x-1-2|\geq \epsilon\Longrightarrow |2\cdot 1-3|=|-1|=1$$
and
$$|x-1|<\delta \Longrightarrow |1-1|=0<\delta$$
By proving the negated statement, ##\displaystyle{\lim_{x \to 1}}(2x-1)\neq 2##
Method 1 makes no sense to me at this time in the morning.

Method 2 is the right approach, but it's not right.

First, it should be ##0 < |x - a| < \delta## and that the ##0 < ## is important. In other words, the limit is independent of ##f(a)##. And, in fact, ##f## may be undefined at ##a##. You cannot choose ##x = 1## when analysing the limit in this case.

Second, at the start of the proof I would say simply that you will show the definition does not hold for ##\epsilon = 1##.

Third, at this point ##\epsilon## should disappear from your working, and be replaced by ##1##. This is similar to the point I made above, where you seem reluctant to abandon the full generality of the definition.

So, the start of the proof should look like:

We show that the defintion of the limit fails for ##\epsilon = 1##. I.e. it is enough to show that:
$$\forall \delta > 0, \exists x: 0 < |x - 1| < \delta \ \text{and} \ |(2x -1) - 2| \ge 1$$That statement alone shows that a) you understand the negation of the limit definition; and b) shows specifically what you have to do in this case.
 
  • Like
Likes docnet
  • #14
A result you can use once you can figure these with a ##\delta -\epsilon## format is that if ##f## is continuous , then ##[x \rightarrow x_0] \rightarrow [ f(x) \rightarrow f(x_0)]##.
 
  • Like
Likes docnet

1. What is real analysis?

Real analysis is a branch of mathematics that deals with the study of real numbers and their properties. It involves the use of rigorous mathematical techniques to analyze the behavior of functions and sequences of real numbers.

2. What does it mean to prove the limit exists?

Proving the limit exists means showing that a function or sequence approaches a specific value as its input or index approaches a certain value. This value is known as the limit and it represents the behavior of the function or sequence near that point.

3. How is the limit of a function or sequence determined?

The limit of a function or sequence is determined by evaluating the behavior of the function or sequence as its input or index approaches the desired value. This can be done through various methods such as direct substitution, algebraic manipulation, or using theorems and properties of limits.

4. Why is proving the limit exists important?

Proving the limit exists is important because it allows us to understand the behavior of functions and sequences in a specific point or interval. This knowledge is crucial in many areas of mathematics and science, such as calculus, physics, and engineering.

5. What are some common techniques used to prove the limit exists?

Some common techniques used to prove the limit exists include the epsilon-delta definition, the squeeze theorem, and the use of limit laws and properties. These techniques involve rigorous mathematical reasoning and logic to show that the limit of a function or sequence exists and has a specific value.

Similar threads

  • Calculus and Beyond Homework Help
Replies
5
Views
1K
  • Calculus and Beyond Homework Help
Replies
2
Views
843
  • Calculus and Beyond Homework Help
Replies
9
Views
2K
  • Calculus and Beyond Homework Help
Replies
19
Views
1K
  • Calculus and Beyond Homework Help
Replies
17
Views
620
Replies
1
Views
631
  • Calculus and Beyond Homework Help
Replies
5
Views
877
  • Calculus and Beyond Homework Help
Replies
16
Views
1K
  • Calculus and Beyond Homework Help
Replies
2
Views
714
  • Calculus and Beyond Homework Help
Replies
2
Views
711
Back
Top